Những câu hỏi liên quan
Bạn Thân Yêu
Xem chi tiết
TRẦN MINH NGỌC
12 tháng 4 2016 lúc 21:47

Giả sử không mất tính tổng quát : a < b < c

=> 1 / a > 1 / b > 1 / c

=> 1 / a + 1 / a + 1 / a > 1 / a + 1 / b + 1 / c > 1 / c + 1 / c + 1 / c

=> 3  .  1/ a   > 4 / 5  > 3   . 1 / c

Đến đây cậu có thể là được rồi

Bình luận (0)
Nguyễn Linh Chi
Xem chi tiết
tth_new
13 tháng 11 2019 lúc 18:43

Bài này cần chú ý: \(\frac{a}{b}+\frac{b}{c}+\frac{c}{a}-3=\frac{\left(a-b\right)^2}{ab}+\frac{\left(a-c\right)\left(b-c\right)}{ac}\)

Và \(\frac{a}{b+c}+\frac{b}{c+a}+\frac{c}{a+b}-\frac{3}{2}=\frac{\left(a-b\right)^2}{\left(a+c\right)\left(b+c\right)}+\frac{\left(a+b+2c\right)\left(a-c\right)\left(b-c\right)}{2\left(a+b\right)\left(b+c\right)\left(c+a\right)}\)

Thêm 3 vào 2 vế ta cần chứng minh:

\(\frac{2}{1-a}+\frac{2}{1-b}+\frac{2}{1-c}\le2\left(\frac{a}{b}+\frac{b}{c}+\frac{c}{a}+\frac{3}{2}\right)\)

\(\Leftrightarrow\frac{a+b+c}{a+b}+\frac{a+b+c}{b+c}+\frac{a+b+c}{c+a}\le\frac{a}{b}+\frac{b}{c}+\frac{c}{a}+\frac{3}{2}\) (chia hai vế cho 2 và chú ý 1 =a + b + c)

\(\Leftrightarrow\frac{3}{2}+\frac{a}{b+c}+\frac{b}{c+a}+\frac{c}{a+b}\le\frac{a}{b}+\frac{b}{c}+\frac{c}{a}\)

\(\Leftrightarrow\frac{a}{b+c}+\frac{b}{c+a}+\frac{c}{a+b}-\frac{3}{2}\le\frac{a}{b}+\frac{b}{c}+\frac{c}{a}-3\)

\(\Leftrightarrow\frac{\left(a-b\right)^2}{\left(a+c\right)\left(b+c\right)}+\frac{\left(a+b+2c\right)\left(a-c\right)\left(b-c\right)}{2\left(a+b\right)\left(b+c\right)\left(c+a\right)}\le\frac{\left(a-b\right)^2}{ab}+\frac{\left(a-c\right)\left(b-c\right)}{ac}\)

\(\Leftrightarrow\left(a-b\right)^2\left(\frac{1}{ab}-\frac{1}{\left(a+c\right)\left(b+c\right)}\right)+\left(\frac{1}{ac}-\frac{a+b+2c}{2\left(a+b\right)\left(b+c\right)\left(c+a\right)}\right)\left(a-c\right)\left(b-c\right)\ge0\)

Quy đồng mỗi cái ngoặc to phía sau là thấy nó > 0:D

Giả sử c = min{a,b,c} như vậy (a-c)(b-c)\(\ge0\) chúng ta có đpcm.

Is that true?

Bình luận (0)
 Khách vãng lai đã xóa
Thắng Nguyễn
13 tháng 11 2019 lúc 20:39

WLOG \(b=mid\left\{a,b,c\right\}\). Áp dụng một bổ đề trong một bài giải của alibaba nguyễn trong câu hỏi của Neet ở học 24. Mọi người có thể tự chứng minh để nhớ lâu hoặc ai cần có thể hỏi ổng

\(\frac{a}{b}+\frac{b}{c}+\frac{c}{a}\ge\frac{a+b}{b+c}+\frac{b+c}{a+b}+1\) với a,b,c>0

Khi đó ta cần chứng minh \(2\left(\frac{a+b}{b+c}+\frac{b+c}{a+b}\right)+2\ge\frac{2a+b+c}{b+c}+\frac{2b+c+a}{c+a}+\frac{2c+a+b}{a+b}\)

\(\Leftrightarrow\frac{a+b}{b+c}+\frac{b+c}{a+b}-\frac{1}{2}\ge\frac{a}{b+c}+\frac{b}{c+a}+\frac{c}{a+b}\)

\(\Leftrightarrow\frac{b}{b+c}+\frac{b}{a+b}-\frac{1}{2}\ge\frac{b}{c+a}\)

\(\Leftrightarrow\frac{\left(a-b\right)\left(b-c\right)\left(a+c+2b\right)}{2\left(a+b\right)\left(b+c\right)\left(c+a\right)}\ge0\)*đúng với \(b=mid\left\{a,b,c\right\}\)*

Bình luận (0)
 Khách vãng lai đã xóa
zZz Cool Kid_new zZz
12 tháng 6 2020 lúc 11:28

Lục vui câu hỏi của cô Chi thấy vài bài ngon mà mấy God dùng đao to vãi :))

\(\frac{1+a}{1-a}=\frac{1-a+2a}{1-a}=1+\frac{2a}{1-a}=1+\frac{2a}{b+c}\)

Khi đó BĐT cần chứng minh tương đương với:

\(3+\frac{2a}{b+c}+\frac{2b}{c+a}+\frac{2c}{a+b}\le2\left(\frac{a}{b}+\frac{b}{c}+\frac{c}{a}\right)\)

\(\Leftrightarrow\frac{a}{b+c}+\frac{b}{c+a}+\frac{c}{a+b}+\frac{3}{2}\le\frac{a}{b}+\frac{b}{c}+\frac{c}{a}\)

\(\Leftrightarrow\frac{ac}{b\left(b+c\right)}+\frac{bc}{a\left(a+b\right)}+\frac{ab}{c\left(c+a\right)}\ge\frac{3}{2}\)

Mặt khác:

\(LHS=\Sigma\frac{ac}{b\left(b+c\right)}=\Sigma\frac{a^2c^2}{abc\left(b+c\right)}\ge\frac{\left(ab+bc+ca\right)^2}{\Sigma abc\left(b+c\right)}=\frac{\left(ab+bc+ca\right)^2}{2abc\left(a+b+c\right)}\)

Ta cần chứng minh \(\frac{\left(ab+bc+ca\right)^2}{2abc\left(a+b+c\right)}\ge\frac{3}{2}\Leftrightarrow\left(ab+bc+ca\right)^2\ge3abc\left(a+b+c\right)\)

Tuy nhiên đây là bổ đề quen thuộc

Vậy ta có đpcm

Bình luận (0)
 Khách vãng lai đã xóa
Shiro Nerin
Xem chi tiết
Phú Quý Lê Tăng
20 tháng 2 2019 lúc 21:05

Ta có: 

\(\frac{1}{a}+\frac{1}{c}=\frac{2}{b}\Leftrightarrow b=\frac{2}{\frac{1}{a}+\frac{1}{c}}=\frac{2ac}{a+c}\)

Thế \(b=\frac{2ac}{a+c}\) vào M, ta được:

 \(M=\frac{a+b}{2a-b}+\frac{c+b}{2c-b}=\frac{a+\frac{2ac}{a+c}}{2a-\frac{2ac}{a+c}}+\frac{c+\frac{2ac}{a+c}}{2c-\frac{2ac}{a+c}}=\frac{1+\frac{2c}{a+c}}{2-\frac{2c}{a+c}}+\frac{1+\frac{2a}{a+c}}{2-\frac{2a}{a+c}}\)

\(M=\frac{\left(a+c\right)+2c}{2\left(a+c\right)-2c}+\frac{\left(a+c\right)+2a}{2\left(a+c\right)-2a}=\frac{a+3c}{2a}+\frac{3a+c}{2c}\)

\(M+2=\frac{a+3c}{2a}+1+\frac{3a+c}{2c}+1=\frac{3a+3c}{2a}+\frac{3a+3c}{2c}=\frac{3}{2}\left(a+c\right)\left(\frac{1}{a}+\frac{1}{c}\right)\)

\(M+2=\frac{3}{2}\left(1+\frac{a}{c}+\frac{c}{a}+1\right)=\frac{3}{2}\left(2+\frac{a}{c}+\frac{c}{a}\right)\)

Xét \(\frac{a}{c}+\frac{c}{a}\ge2\Leftrightarrow...\)(bạn tự biến đổi tương đương để chứng minh nó nhé)

(ĐK xảy ra dấu "=": a=c)

Do đó \(M+2=\frac{3}{2}\left(1+\frac{a}{c}+\frac{c}{a}+1\right)=\frac{3}{2}\left(2+\frac{a}{c}+\frac{c}{a}\right)\ge\frac{3}{2}\left(2+2\right)=6\Leftrightarrow M\ge4\)

Vậy GTNN của \(M=4\)khi \(a=c\Leftrightarrow\frac{2}{b}=\frac{2}{a}\Leftrightarrow b=a=c\)

Chúc bạn học tốt!

P/S: bài này khó thật đấy! Mình chuyên toán 9 mà giải hết nửa tiếng mới xong :D!

Bình luận (0)
Nữ Thám Tử THông Minh
Xem chi tiết
FA là tao
Xem chi tiết
Nguyễn Anh Quân
7 tháng 1 2018 lúc 21:00

b, Có: a/b < c/d => ad < bc

 Xét a.(b+d)-b.(a+c) = ab+ad-ba-bc = ad-bc < 0

=> a.(b+d) < b.(a+c)

=> a/b < a+c/b+d

c, Đề phải là cho a+b+c = 2016 chứ bạn

Có : A = a/a+b+c-c + b/a+b+c-a + c/a+b+c-b = a/a+b + b/b+c + c/c+a

Vì a,b,c thuộc Z+ nên a/a+b > 0 ; b/b+c > 0 ; c/c+a > 0

=> A > a/a+b+c + b/a+b+c + c/a+b+c = 1

Lại có : a < a+b ; b < b+c ; c < c+a => 0 < a/a+b < a ; 0 < b/b+c < 1 ; 0 < c/c+a < 1

=> A < a+c/a+b+c + b+a/a+b+c + c+b/a+b+c = 2

=> 1 < A < 2

=> A ko phải là số tự nhiên

Tk mk nha

Bình luận (0)
Nguyễn Bảo Chi
7 tháng 1 2018 lúc 20:59

a,ÁP DỤNG TÍNH CHẤT DÃY TỈ SỐ BẰNG NHAU.

TA CÓ:\(\frac{a}{b}\)=\(\frac{b}{c}\)=\(\frac{c}{d}\)=\(\frac{d}{e}\)=>\(\frac{2a^2}{2b^2}\)=\(\frac{3b^2}{3c^2}\)=\(\frac{4c^2}{4d^2}\)=\(\frac{5d^2}{5e^2}\)=\(\frac{2a^2+3b^2+4c^2+5d^2}{2b^2+3c^2+4d^2+5e^2}\)(đfcm)

Bình luận (0)
Nguyễn Sỹ Dũng
7 tháng 1 2018 lúc 21:04

Trung hop roi,to dang FA

Bình luận (0)
Nguyễn Thị Hoài An
Xem chi tiết
Lê Thảo Vy
Xem chi tiết
Nguyễn Quang Huy
15 tháng 10 2017 lúc 20:15

dễ ợt mày ngu thế

Bình luận (0)
Đoàn Phương Liên
Xem chi tiết
Nguyễn Linh Chi
6 tháng 7 2019 lúc 11:31

Em tham khảo link:Câu hỏi của Conan Kudo - Toán lớp 8 - Học toán với OnlineMath

Bình luận (0)
ღ๖ۣۜLinh
6 tháng 7 2019 lúc 11:33

Ta có bổ đề

\(\frac{1}{a}+\frac{1}{b}+\frac{1}{c}=0\)\(\Leftrightarrow\frac{1}{a^3}+\frac{1}{b^3}+\frac{1}{c^3}=\frac{3}{abc}\)

ÁP DỤNG BỔ ĐỀ VÀO P ta có

\(P=\frac{bc}{a^2}+\frac{ca}{b^2}+\frac{ab}{c^2}=abc\left(\frac{1}{a^3}+\frac{1}{b^3}+\frac{1}{c^3}\right)\)

\(=abc.\frac{3}{abc}=3\)

Vậy P=3

Bình luận (0)
Tình Nguyễn Thị
Xem chi tiết
Bui Huyen
18 tháng 2 2019 lúc 19:57

\(\frac{a}{2}+\frac{b}{3}=\frac{a+b}{5}\Leftrightarrow\frac{3a+2b}{6}=\frac{a+b}{5}\\ \Rightarrow15a+10b=6a+6b\Rightarrow9a+4b=0\)

mà a,b là số tự nhiên nên \(a,b\ge0\)

nên \(9a+4b\ge0\)

dấu bằng xảy ra khi a=b=0

Bình luận (0)
Bui Huyen
18 tháng 2 2019 lúc 20:05

mk làm sai nha bạn

sr bạn

Bình luận (0)